В современном мире многие люди интересуются наукой и техникой и пытаются хотя бы в общих чертах понять, как работают вещи, которые их окружают. Благодаря этому стремлению к просвещению существует научно-просветительская литература и сайты, подобные Гиктаймсу. А поскольку читать и воспринимать ряды формул большинству людей затруднительно, то излагаемые в подобных изданиях теории неизбежно подвергаются значительному упрощению в попытке донести до читателя «суть» идеи с помощью простого и понятного объяснения которое легко воспринять и запомнить. К сожалению, некоторые из подобных «простых объяснений» являются в корне неверными, но при этом оказываются настолько «очевидными», что не подвергаясь особому сомнению начинают кочевать из одного издания в другое и нередко становятся доминирующей точкой зрения, несмотря на свою ошибочность.

В качестве одного из примеров попробуйте ответить на простой вопрос: «откуда возникает подъемная сила в крыле самолета»?



Если в Вашем объяснении фигурируют «разная длина верхней и нижней поверхности крыла», «разная скорость потока воздуха на верхней и нижней кромках крыла» и «закон Бернулли», то я вынужден Вам сообщить, что Вы скорее всего стали жертвой популярнейшего мифа, который преподают порою даже в школьной программе.

Давайте для начала напомним, о чем идет речь



Объяснение подъемной силы крыла в рамках мифа выглядит следующим образом:
  1. Крыло имеет несимметричный профиль снизу и сверху
  2. Непрерывный поток воздуха разделяется крылом на две части, одна из которых проходит над крылом, а другая под ним
  3. Мы рассматриваем ламинарное обтекание, в котором поток воздуха плотно прилегает к поверхности крыла
  4. Поскольку профиль несимметричен, то для того чтобы снова сойтись за крылом в одной точке «верхнему» потоку нужно проделать больший путь, чем «нижнему», поэтому воздуху над крылом приходится двигаться с большей скоростью чем под ним
  5. Согласно закону Бернулли статическое давление в потоке уменьшается с ростом скорости потока, поэтому в потоке над крылом статическое давление будет ниже
  6. Разница давлений в потоке под крылом и над ним и составляет подъемную силу

А для демонстрации этой идеи достаточно простого гибкого и легкого листа бумаги. Берем лист, подносим его ко рту, и дуем над ним чтобы создать модель в которой поток воздуха над листом бумаги движется быстрее чем под ним. И вуаля — с первой или второй попытки лист бумаги презрев тяготение действительно поднимается под действием подъемной силы вверх. Теорема доказана!

… или все-таки нет?..

Существует история (я правда не знаю насколько она правдива), что одним из первых людей предложивших, подобную теорию был не кто иной, как сам Альберт Эйнштейн. Согласно этой истории в 1916 году он написал соответствующую статью и на её основе предложил свою версию «идеального крыла», которое, по его мнению, максимизировало разницу скоростей над крылом и под ним, и в профиль выглядело примерно вот так:



В аэродинамической трубе продули полноценную модель крыла с этим профилем, но увы — её аэродинамические качества оказались на редкость плохими. В отличие — парадоксально! — от многих крыльев с идеально симметричным профилем, в которых путь воздуха над крылом и под ним должен был быть принципиально одинаков. В рассуждениях Эйнштейна явно что-то было неправильно. И вероятно наиболее явным проявлением этой неправильности было то что некоторые пилоты в качестве акробатического трюка стали летать на своих самолетах вверх ногами. У первых самолетов, которые пробовали перевернуться в полете, возникали проблемы с топливом и маслом, которое не текло туда, куда нужно, и вытекало там, где не нужно, но после того, как в 30-х годах прошлого века энтузиастами аэробатики были созданы топливные и масляные системы, способные работать длительное время в перевернутом положении, полет «вверх ногами» стал обычным зрелищем на авиашоу. В 1933, к примеру, один американец и вовсе совершил полет вверх ногами из Сан-Диего в Лос-Анджелес. Каким-то волшебным образом перевернутое крыло по-прежнему генерировало подъемную силу, направленную вверх.



Посмотрите на эту картинку — на ней изображен самолет, аналогичный тому, на котором был установлен рекорд полета в перевернутом положении. Обратите внимание на обычный профиль крыла (Boeing-106B airfoil) который, согласно приведенным выше рассуждениям, должен создавать подъемную силу от нижней поверхности к верхней.

Итак, у нашей простой модели подъемной силы крыла есть некоторые трудности, которые можно в целом свести к двум простым наблюдениям:
  1. Подъемная сила крыла зависит от его ориентации относительно набегающего потока воздуха — угла атаки
  2. Симметричные профили (в том числе и банальный плоский лист фанеры) тоже создают подъемную силу

В чем же причина ошибки? Оказывается, что в приведенном в начале статьи рассуждении совершенно неверен (и вообще говоря, просто взят с потолка) пункт №4. Визуализация потока воздуха вокруг крыла в аэродинамической трубе показывает, что фронт потока, разделенный на две части крылом, вовсе не смыкается обратно за кромкой крыла.



Проще говоря, воздух «не знает», что ему нужно двигаться с какой-то определенной скоростью вокруг крыла, чтобы выполнить какое-то условие, которое нам кажется очевидным. И хотя скорость потока над крылом действительно выше, чем под ним, это является не причиной образования подъемной силы а следствием того, что над крылом существует область пониженного давления, а под крылом — область повышенного. Попадая из области нормального давления в разреженную область, воздух разгоняется перепадом давлений, а попадая в область с повышенным давлением — тормозится. Важный частный пример столь «не-бернуллевского» поведения наглядно демонстрируют экранопланы: при приближении крыла к земле его подъемная сила возрастает (область повышенного давления поджимается землей), тогда как в рамках «бернуллевских» рассуждений крыло на пару с землей формируют нечто вроде сужающегося тоннеля что в рамках наивных рассуждений должно было бы разгонять воздух и притягивать за счет этого крыло к земле подобно тому, как это делается в схожих по смыслу рассуждениях о «взаимном притяжении проходящих на параллельных курсах пароходах». Причем в случае экраноплана ситуация во многом даже хуже, поскольку одна из «стенок» этого тоннеля движется с высокой скоростью навстречу крылу, дополнительно «разгоняя» тем самым воздух и способствуя еще большему снижению подъемной силы. Однако реальная практика «экранного эффекта» демонстрирует прямо противоположную тенденцию, наглядно демонстрируя опасность логики рассуждений о подъемной силе построенных на наивных попытках угадать поле скоростей потока воздуха вокруг крыла.

Как это ни странно, значительно более приближенное к истине объяснение дает другая неверная теория подъемной силы, отвергнутая еще в XIX веке. Сэр Исаак Ньютон предполагал, что взаимодействие объекта с набегающим воздушным потоком можно моделировать, предположив, что набегающий поток состоит из крошечных частиц, ударяющихся об объект и отскакивающих от него. При наклонном расположении объекта относительно набегающего потока частицы будут преимущественно отражаться объектом вниз и в силу закона сохранения импульса при каждом отклонении частицы потока вниз объект будет получать импульс движения вверх. Идеальным крылом в подобной модели был бы плоский воздушный змей, наклоненный к набегающему потоку:



Подъемная сила в этой модели возникает за счет того, что крыло направляет часть воздушного потока вниз, это перенаправление требует приложения определенной силы к потоку воздуха, а подъемная сила является соответствующей силой противодействия со стороны воздушного потока на крыло. И хотя исходная «ударная» модель вообще говоря неверна, в подобной обобщенной формулировке это объяснение действительно верно. Любое крыло работает за счет того, что отклоняет часть набегающего потока воздуха вниз и это, в частности, объясняет, почему подъемная сила крыла пропорциональна плотности потока воздуха и квадрату его скорости. Это дает нам первое приближение к правильному ответу: крыло создает подъемную силу потому что линии тока воздуха после прохождения крыла в среднем оказываются направлены вниз. И чем сильнее мы отклоняем поток вниз (например увеличивая угол атаки) — тем подъемная сила оказывается больше.



Немного неожиданный результат, правда? Однако он пока никак не приближает нас к пониманию того, почему воздух после прохождения крыла оказывается движущимся вниз. То, что Ньютоновская ударная модель неверна, было показано экспериментально опытами, которые продемонстрировали что реальное сопротивление потока ниже, чем предсказывает Ньютоновская модель, а генерируемая подъемная сила — выше. Причиной этих расхождений является то, что в модели Ньютона частички воздуха никак не взаимодействуют друг с другом, тогда как реальные линии тока не могут пересекать друг друга, так как это показано на рисунке выше. «Отскакивающие» под крылом вниз условные «частички воздуха» сталкиваются с другими и начинают «отталкивать» их от крыла еще до того, как они с ним столкнутся, а частички воздушного тока, оказавшиеся над крылом, «выпихивают» частички воздуха, расположенные ниже, в пустое пространство, остающееся за крылом:



Говоря другими словами, взаимодействие «отскочившего» и «набегающего» потоков создает под крылом область высокого давления (красную), а «тень», пробиваемая крылом в потоке, образует область низкого давления (синюю). Первая область отклоняет поток под крылом вниз еще до того, как этот поток соприкоснется с его поверхностью, а вторая заставляет поток над крылом изгибаться вниз, хотя он с крылом не соприкасался вообще. Совокупное давление этих областей по контуру крыла, собственно, и образует в итоге подъемную силу. При этом интересный момент состоит в том, что неизбежно возникающая перед крылом область высокого давления у правильно спроектированного крыла соприкасается с его поверхностью лишь по небольшому участку в передней кромке крыла, тогда как область высокого давления под крылом и область низкого давления над ним соприкасаются с крылом на значительно большой площади. В результате подъемная сила крыла формируемая двумя областями вокруг верхней и нижней поверхностей крыла может быть намного больше, чем сила сопротивления воздуха, которую обеспечивает воздействие области высокого давления, расположенной перед передней кромкой крыла.



Поскольку наличие областей разного давления изгибает линии тока воздуха, то часто удобно определять эти области именно по этому изгибу. К примеру, если линии тока над крылом «загибаются вниз», то в этой области существует градиент давления направленный сверху вниз. И если на достаточно большом удалении над крылом давление является атмосферным, то по мере приближения к крылу сверху вниз давление должно падать и непосредственно над крылом оно окажется ниже атмосферного. Рассмотрев аналогичное «искривление вниз», но уже под крылом, мы получаем, что если начать с достаточно низкой точки под крылом, то, приближаясь к крылу снизу вверх, мы придем в область давления, которое будет выше атмосферного. Аналогичным образом «расталкивание» линий тока перед передней кромкой крыла соответствует существованию перед этой кромкой области повышенного давления. В рамках подобной логики можно сказать, что крыло создает подъемную силу, изгибая линии тока воздуха вокруг крыла. Поскольку линии тока воздуха как бы «прилипают» к поверхности крыла (эффект Коанда) и друг к другу, то, изменяя профиль крыла, мы заставляем воздух двигаться вокруг него по искривленной траектории и формировать в силу этого нужный нам градиент давлений. К примеру, для обеспечения полета вверх ногами достаточно создать нужный угол атаки, направив нос самолета в сторону от земли:



Снова немного неожиданно, правда? Тем не менее это объяснение уже ближе к истине, чем исходная версия «воздух ускоряется над крылом, потому что над крылом ему нужно пройти большее расстояние, чем под ним». Кроме того, в его терминах легче всего понять явление, которое называется «срывом потока» или «сваливанием самолета». В нормальной ситуации увеличивая угол атаки крыла мы увеличиваем тем самым искривление воздушного потока и соответственно подъемную силу. Ценою за это является увеличение аэродинамического сопротивления, поскольку область низкого давления постепенно смещается из положения «над крылом» в положение «слегка за крылом» и соответственно начинает притормаживать самолет. Однако после некоторого предела ситуация неожиданно резко изменяется. Синяя линия на графике — коэффициент подъемной силы, красная — коэффициент сопротивления, горизонтальная ось соответствует углу атаки.



Дело в том, что «прилипаемость» потока к обтекаемой поверхности ограничена, и если мы попытаемся слишком сильно искривить поток воздуха, то он начнет «отрываться» от поверхности крыла. Образующаяся за крылом область низкого давления начинает «засасывать» не поток воздуха, идущий с ведущей кромки крыла, а воздух из области оставшейся за крылом, и подъемная сила генерируемая верхней частью крыла полностью или частично (в зависимости от того, где произошел отрыв) исчезнет, а лобовое сопротивление увеличится.



Для обычного самолета сваливание — это крайне неприятная ситуация. Подъемная сила крыла уменьшается с уменьшением скорости самолета или уменьшением плотности воздуха, а кроме того поворот самолета требует большей подъемной силы, чем просто горизонтальный полет. В нормальном полете все эти факторы компенсируют именно выбором угла атаки. Чем медленнее летит самолет, чем менее плотный воздух (самолет забрался на большую высоту или садится в жаркую погоду) и чем круче поворот, тем больше приходится делать этот угол. И если неосторожный пилот переходит определенную черту, то подъемная сила упирается в «потолок» и становится недостаточной для удержания самолета в воздухе. Добавляет проблем и увеличившееся сопротивление воздуха, которое ведет к потере скорости и дальнейшему снижению подъемной силы. А в результате самолет начинает падать — «сваливается». Попутно могут возникнуть проблемы с управлением из-за того, что подъемная сила перераспределяется по крылу и начинает пытаться «повернуть» самолет или управляющие поверхности оказываются в области сорванного потока и перестают генерировать достаточное управляющее усилие. А в крутом повороте, к примеру, поток может сорвать лишь с одного крыла, в результате чего самолет начнет не просто терять высоту, но и вращаться — войдет в штопор. Сочетание этих факторов остается одной из нередких причин авиакатастроф. С другой стороны, некоторые современные боевые самолеты специально проектируются таким специальным образом, чтобы сохранять управляемость в подобных закритических режимах атаки. Это позволяет подобным истребителям при необходимости резко тормозить в воздухе. Иногда это используется для торможения в прямолинейном полете, но чаще востребовано в виражах, поскольку чем меньше скорость, тем меньше при прочих равных радиус поворота самолета. И да-да, Вы угадали — именно это та самая «сверхманевренность», которой заслуженно гордятся специалисты проектировавшие аэродинамику отечественных истребителей 4 и 5 поколений.



Однако мы пока так и не ответили на основной вопрос: откуда, собственно, возникают области повышенного и пониженного давления вокруг крыла в набегающем потоке воздуха? Ведь оба явления («прилипание потока к крылу» и «над крылом воздух движется быстрее»), которыми можно объяснить полет, являются следствием определенного распределения давлений вокруг крыла, а не его причиной. Но почему формируется именно такая картина давлений, а не какая-то другая?



К сожалению, ответ на этот вопрос уже неизбежно требует привлечения математики. Давайте представим себе, что наше крыло является бесконечно длинным и одинаковым по всей длине, так что движение воздуха вокруг него можно моделировать в двумерном срезе. И давайте предположим, для начала, что в роли нашего крыла выступает… бесконечно длинный цилиндр в потоке идеальной жидкости. В силу бесконечности цилиндра такую задачу можно свести к рассмотрению обтекания круга в плоскости потоком идеальной жидкости. Для столь тривиального и идеализированного случая существует точное аналитическое решение, предсказывающее, что при неподвижном цилиндре общее воздействие жидкости на цилиндр будет нулевым.



А теперь давайте рассмотрим некое хитрое преобразование плоскости на себя, которое математики называют конформным отображением. Оказывается можно подобрать такое преобразование, которое с одной стороны сохраняет уравнения движения потока жидкости, а с другой трансформирует круг в фигуру, имеющую похожий на крыло профиль. Тогда трансформированные тем же самым преобразованием линии тока жидкости для цилиндра становятся решением для тока жидкости вокруг нашего импровизированного крыла.



Наш исходный круг в потоке идеальной жидкости имеет две точки, в которых линии тока соприкасаются с поверхностью круга, и следовательно те же две точки будут существовать и на поверхности профиля после применения к цилиндру преобразования. И в зависимости от поворота потока относительно исходного цилиндра («угла атаки») они будут располагаться в разных местах поверхности сформированного «крыла». И почти всегда это будет означать, что часть линий тока жидкости вокруг профиля должна будет огибать заднюю, острую кромку крыла, как показано на картинке выше.

Это потенциально возможно для идеальной жидкости. Но не для реальной.

Наличие в реальной жидкости или газе даже небольшого трения (вязкости) приводит к тому, что поток подобный изображенному на картинке немедленно нарушается — верхний поток будет сдвигать точку где линия тока соприкасается с поверхностью крыла до тех, пор пока она не окажется строго на задней кромке крыла (постулат Жуковского-Чаплыгина, он же аэродинамическое условие Кутты). И если преобразовать «крыло» обратно в «цилиндр», то сдвинувшиеся линии тока окажутся примерно такими:



Но если вязкость жидкости (или газа) очень мала, то получившееся подобным путем решение должно подходить и для цилиндра. И оказывается, что такое решение действительно можно найти, если предположить, что цилиндр вращается. То есть физические ограничения, связанные с перетоком жидкости вокруг задней кромки крыла приводят, к тому, что движение жидкости из всех возможных решений будет стремиться прийти к одному конкретному решению, в котором часть потока жидкости вращается вокруг эквивалентного цилиндра, отрываясь от него в строго определенной точке. А поскольку вращающийся цилиндр в потоке жидкости создает подъемную силу, то ее создает и соответствующее крыло. Компонент движения потока соответствующий этой «скорости вращения цилиндра» называется циркуляцией потока вокруг крыла, а теорема Жуковского говорит о том, что аналогичную характеристику можно обобщить для произвольного крыла, и позволяет количественно рассчитывать подъемную силу крыла на ее основе. В рамках этой теории подъемная сила крыла обеспечивается за счет циркуляции воздуха вокруг крыла, которая порождается и поддерживается у движущегося крыла указанными выше силами трения, исключающими переток воздуха вокруг его острой задней кромки.

Удивительный результат, не правда ли?

Описанная теория конечно сильно идеализирована (бесконечно длинное однородное крыло, идеальный однородный несжимаемый поток газа / жидкости без трения вокруг крыла), но дает довольно точное приближение для реальных крыльев и обычного воздуха. Только не воспринимайте в ее рамках циркуляцию как свидетельство того, что воздух действительно вращается вокруг крыла. Циркуляция — это просто число, показывающее, насколько должен отличаться по скорости поток на верхней и нижней кромках крыла, чтобы решение движений потока жидкости обеспечило отрыв линий тока строго на задней кромке крыла. Не стоит также воспринимать «принцип острой задней кромки крыла» как необходимое условие для возникновения подъемной силы: последовательность рассуждений вместо этого звучит как «если у крыла острая задняя кромка, то подъемная сила формируется так-то».

Попробуем подытожить. Взаимодействие воздуха с крылом формирует вокруг крыла области высокого и низкого давления, которые искривляют воздушный поток так, что он огибает крыло. Острая задняя кромка крыла приводит к тому, что в идеальном потоке из всех потенциальных решений уравнений движения реализуется только одно конкретное, исключающее переток воздуха вокруг острой задней кромки. Это решение зависит от угла атаки и у обычного крыла имеет область пониженного давления над крылом и область повышенного давления — под ним. Соответствующая разница давлений формирует подъемную силу крыла, заставляет воздух двигаться быстрее над верхней кромкой крыла и замедляет воздух под нижней. Количественно подъемную силу удобно описывать численно через эту разницу скоростей над крылом и под ним в виде характеристики, которая называется «циркуляцией» потока. При этом в соответствии с третьим законом Ньютона действующая на крыло подъемная сила означает, что крыло отклоняет вниз часть набегающего воздушного потока — для того, чтобы самолет мог лететь, часть окружающего его воздуха должна непрерывно двигаться вниз. Опираясь на этот движущийся вниз поток воздуха самолет и «летит».

Простое же объяснение с «воздухом, которому нужно пройти более длинный путь над крылом, чем под ним» — неверно.
Поделиться с друзьями
-->

Комментарии (172)


  1. avost
    24.08.2016 09:37
    +9

    >Если в Вашем объяснении фигурируют «разная скорость потока на верхней и нижней кромках крыла» и «закон Бернулли», то я вынужден Вам сообщить, что Вы скорее всего стали жертвой популярнейшего мифа

    >Указанная разница скоростей равносильна тому что над крылом (при достаточном угле атаки) возникает область пониженного давления, а под крылом — область пониженного

    Но ведь вы в самом начале постулировали, что если в объяснении фигурирует «разная скорость потока», то это миф. Или вы имеете в виду, что у вас «другая» «разная скорость потоков»? Та, первая «разная скорость потоков» неправильная и не создаёт подъёмной силы, эта вторая «разная скорость потоков» правильная и силу создаёт? :)


    1. 0serg
      24.08.2016 09:39
      -1

      Я не нашёл способа как это более удачно выразить. Принципиальных моментов два: во-первых разница в скоростях — это скорее следствие разницы в давлениях, а не наоборот, а во-вторых объяснение того что скорость воздуха над крылом больше из-за того что воздуху там требуется пройти больший путь — неверно.


      1. nkie
        24.08.2016 10:54
        +3

        Все хорошо в статье кроме последнего абзаца, из которого создается впечатление что подъемная сила как раз и создается разностью скоростей, что вы в самом начале пытаетесь опровергнуть и люди запутываются. Я бы на вашем месте как-нибудь этот абзац все-таки перефразировал.


        1. 0serg
          24.08.2016 11:18
          -1

          Спасибо! Попробовал перефразировать


      1. avost
        24.08.2016 11:29

        Неверно про путь, да. А вот про разницу в скоростях совсем неочевидно. Ну, и манипуляции с углом атаки не всегда корректны. Там даже картинка есть, с распределением давления традиционного профиля при нулевом угле атаки. Разница давлений присутствует.


      1. NIN
        24.08.2016 12:38
        +2

        > Оказывается что в приведенном в начале статьи рассуждении совершенно неверен (и вообще говоря просто взят с потолка) пункт №4.

        Всё-таки пункт №4 верен, так как из ламинарности обтекания (№3) и разности длины путей действительно следует различие в скорости воздуха.(в этой модели воздух — идеальная жидкость)

        Вы же пытаетесь разоблачить этот миф рассматривая заведомо неламинарный режим.

        > Ведь оба явления («прилипание потока к крылу» и «над крылом воздух движется быстрее») которыми можно объяснить полет являются следствием определенного распределения давлений вокруг крыла, а не его причиной.

        Принципиальность момента относительно «причинности» явления не понятен, это всего лишь Ваш порядок рассуждений. В уравнениях движения жидкости разность давлений определяет изменение скорости или наоборот?


        1. 0serg
          24.08.2016 12:45

          Всё-таки пункт №4 верен, так как из ламинарности обтекания (№3) и разности длины путей действительно следует различие в скорости воздуха


          Только если сделать дополнительно предположение что время в пути должно быть одинаково. А оно не верно.

          Вы же пытаетесь разоблачить этот миф рассматривая заведомо неламинарный режим.


          Ознакомьтесь, пожалуйста, с видео. Там изображен ламинарный режим обтекания.

          Принципиальность момента относительно «причинности» явления не понятен, это всего лишь Ваш порядок рассуждений


          Я согласен с тем что скорость и давление связаны взаимно-однозначно. Но представьте себе что мы взяли тележку и толкнули ее под гору. Скорость тележки и высота на которой она находится тоже связаны взаимно-однозначным соотношением. Поэтому формально можно сказать как «скорость тележки увеличивается за счет того что она катится под гору и теряет высоту» так и «тележка катится с горы потому что скорость ее увеличивается». Но субъективно мне кажется то первая формулировка лучше отражает происходящее.


          1. NIN
            24.08.2016 13:08

            Одинаковость времени следует из того, что жидкость не может накапливаться(сколько натекло на крыло, столько и вытекло за крылом). В противном случае, так как в оба пути течёт постоянно одинаковый поток жидкости, а выходят они «в разное время», жидкость должна куда-то исчезнуть(на выходе, далеко за крылом, поток также везде одинаков).


            1. MoonGoose
              24.08.2016 13:20

              Nope. Если мы рассматриваем несжимаемую жидкость – да. Но принять воздух несжимаемой жидкостью – это верх нелогичных допущений.

              Поэтому, в силу того, что поток постоянен, а путь сверху длиннее (а ещё из-за того, что воздух не может оторваться от крыла и в любом случае вынужден его обтекать), получаем давление сверху меньше, чем снизу, а потом уже поток сверху начинает ускоряться из-за разницы давлений.


              1. NIN
                24.08.2016 13:41

                > Nope. Если мы рассматриваем несжимаемую жидкость – да. Но принять воздух несжимаемой жидкостью – это верх нелогичных допущений.

                Согласен, требуется несжимаемость жидкости. Тем не менее, именно это предполагается в данной модели (косвенно тем, что используется уравнение Бернулли, а также в статье:

                > Описанная теория конечно сильно идеализирована (бесконечно длинное однородное крыло, идеальный однородный несжимаемый поток газа / жидкости без трения вокруг крыла), но дает довольно точное приближение для реальных крыльев и обычного воздуха. )


                1. encyclopedist
                  24.08.2016 14:09
                  +3

                  Сжимаемость тут ни при чем.


              1. ivan01
                24.08.2016 14:33

                это верх нелогичных допущений

                Спасибо, посмеялся! Воздух принимается несжимаемым в крайне широком спектре задачь, по причине того, что ro*V^2/2 << Patm, в общем давление торможения много меньше атмосферного давления на всех скоростях вплоть до 0.8 Маха где-то.


                1. Rzzzz
                  25.08.2016 07:46

                  > Воздух принимается несжимаемым в крайне широком спектре задачь

                  Но воздух сжимаем! Это ж газ все-таки.


                  1. ivan01
                    25.08.2016 11:32

                    Несжимаем, пока мы его не сжимаем, а не сжимаем мы его пока скорость течения маленькая (меньше 0.8 маха). Вода тоже сжимаемая, захотите смоделировать взрыв глубинной бомбы, придется считать сжимаемой.
                    Изменением параметров газа за счет сжимания на 0.01% можно принебречь, разве нет?


                    1. MoonGoose
                      25.08.2016 13:07
                      +1

                      Для своего интереса: а как насчёт расширяемости? С сжимаемостью понятно: слишком мала скорость, а что насчёт расширения в условиях понижения давления (тот же поток над крылом)?


                      1. ivan01
                        25.08.2016 13:21

                        Тоже абсолютно та же физика, покааааа, пока абсолютное давление не падает до давления насыщенного пара, тогда начинается кавитация, но для моделирования обтекания крыла водой это небольшая помеха. Вообще вода лучше т.к.кинематическая вязкость отличается на порядок и проще выдержать подобие, нужны и масшатбы меньше и скорости.


                    1. Rzzzz
                      25.08.2016 16:09

                      Что-то вы прямо поколебали мои познания. А в неподвижном воздухе как тогда? Давление если растет он тоже не сжимается?


                      1. ivan01
                        25.08.2016 17:01

                        Сжимается. Только чтобы значительно сжать воздух движением надо ооооочень быстрое движение. Есть вам разница в расчете между плотностью 1 кг/м^3 и 1.01 кг/м^3? Нету, почти всегда.


                1. Miitall
                  26.08.2016 12:39

                  Если я правильно помню, то до принято полагать несжимаемым до 0,3 Мах. Выводится из закона Бернулли, появляется отношение квадрата скрости к скорости распространиения возмущений (aka скорость звука)


            1. encyclopedist
              24.08.2016 13:41
              +2

              Это никак не следует. Представьте 2 трубы одинаковой длины, но одну с сужением, а другую с расширением, и запустим по ним одинаковый поток. По трубе с расширением частицы жидкости будут затрачивать больше времени на прохождение трубы.


              1. NIN
                24.08.2016 14:11

                Насчёт примера с трубами согласен, но мой комментарий был не о всех случаях, я для примера с крылом.
                Далеко за крылом воздух однороден и идентичен воздуху далеко перед крылом, т.е. в этом случае трубы должны обратно вернуть своё первоначальное сечение.
                Представьте что поток воздуха дует не на перед крыла, а в зад(обратим скорость), ситуация должна быть такой же.
                В трубах с одинаковыми входным и выходным сечениями время также одинаково.


                1. encyclopedist
                  24.08.2016 14:14
                  +1

                  Нет. Я имел ввиду трубы, которые имеют сужение или расширение в середине, а на выходе снова возвращаются к такому же сечению.


                  1. NIN
                    24.08.2016 19:42

                    Понял, виноват, согласен.


                  1. VioletGiraffe
                    25.08.2016 18:47

                    Тогда я не понял. Расход = скорость * площадь сечения. Если на входе расход одинаковый, то и на выходе одинаковый. А т. к. сечение на входе и на выходе одинаковое, то и скорость на выходе должна быть одинаковой, разве нет?


                    1. encyclopedist
                      25.08.2016 18:50

                      Да. Но поскольку в середине сечение разное, то скороть будет разная, и значит время от входа до выхода будет разное.


            1. 0serg
              24.08.2016 13:54
              +2

              Представьте что у Вас есть труба с жидкостью сечением 1 метр квадратный на входе и на выходе.
              Представьте себе что на вход подается вода со скоростью 1 метр в секунду.
              Вы совершенно правильно говорите, что в силу того что вода не может накапливаться в трубе из этого будет автоматически следовать что в выходном сечении мы должны будем увидеть тот же расход воды. В нашем случае сечение одинаково, следовательно и скорость воды там будет 1 метр в секунду.

              Но следует ли из этого что время в пути через трубу будет постоянной величиной? Очевидно что нет. Если мы сузим середину трубы, то скорость потока там вырастет (меньшее сечение при том же расходе воды) и время в пути через трубу уменьшится. А если мы сделаем трубу, скажем, шире или просто тупо длиннее, то время в пути через трубу можно сколь угодно увеличить.

              * уже ответили пока писал комментарий


              1. NIN
                24.08.2016 19:41
                +1

                Вынужден признать свою ошибку, равенство времён действительно ниоткуда не следует. Спасибо за разоблачение!


      1. dobrobelko
        24.08.2016 15:43

        Слишком много шума не по делу. С экранным эффектом тоже мимо — он возникает из-за пониженного индуктивного сопротивления, создаваемого завихрениями на законцовках крыльев.


        1. 4ebriking
          24.08.2016 18:38
          +1

          Э-э-э… нет. Есть две версии экранного эффекта — Липпиша и Алексеева. Соотв у первого экран на высоте примерно равной (полу)размаху крыла (давим поверхностью индукцию), второй — на высоте примерно хорды (создаём за счёт поверхности подпор).
          Липпиш
          Алексеев
          И споры мэтров (и далее вниз от дилетантов к профанам) по этому поводу не утихают с тех самых пор, оставаясь, впрочем где-то на обочине «большой» аэродинамики, которая от этих экранов бежит как чёрт от ладана (даже вертолёты).


          1. dobrobelko
            24.08.2016 20:50

            Что — нет? Первый вариант — как раз который я назвал, снижение индуктивного сопротивления, начинается на расстоянии размаха крыльев. Не эксперт в аэродинамике, в штатах пилотов учат только первому варианту. Когда учился по Part 61 видел только его, спросил знакомых их ERAU и University of North Dakota — у них то же самое.


            1. encyclopedist
              25.08.2016 02:04

              Обычные самолёты и вертолёты используют первый, поэтому его и учат. А экранопланы используют второй тоже.


            1. 4ebriking
              25.08.2016 10:00
              +1

              «нет» — о том, что ситуация ЗНАЧИТЕЛЬНО сложнее и сводить её только к уменьшению индукции на законцовках (кстати индуктивное сопротивление есть и у бесконечного крыла) — суть махровый редукционизм и, в общем, профанация, хотя, для пилотов и сгодится — им аэродинамику надо знать сильно «в общих чертах», т.к. за них все эти тонкости конструкторы уже решили. Напр. неучёт второго при конструировании даже сухопутного самолёта (допустим, сделать ему очень короткие ноги), или низкоплана-амфибии по типу Бе-103, или экранолёта-экраноплана, или противокорабельной крылатой ракеты — чреват итоговой общей неудачей…

              Аэрогидродинамика — наука большая, очень большая и реально сложная практически в каждом своём подразделе — не надо про неё говорить что «много шума из ничего».


              1. dobrobelko
                25.08.2016 10:11

                То есть FAA занимается профанацией? Ок.
                Опять общие фразы о том как все сложно и никакой конкретики. Привести примеры «сухопутных самолетов с короткими ногами» или сами найдете?
                Во-первых, не нужно мешать аэро- и гидродинамику в одну кучу. Во-вторых, много шума из ничего было сказано о якобы развенчании мифа о том как образуется подъемная сила.


                1. 4ebriking
                  25.08.2016 11:19

                  Я вижу вы передёргиваете? Ну ок, приведите pls пример «коротконого» самолёта (что бы высота ног при отрыве была меньше САХ), спроектированного без учёта экранных эффектов. (Рутана просьба не предлагать, ибо не дано знать что он там учитывал, а что — нет).
                  FAA прекрасно знает, что кому знать надо, а что — излишне, пилоты и так загружены знаниями под пробку и давать им (а значит и требовать от них) то, с чем они заведомо никогда не столкнутся в практике (ибо об этом позаботились констуктора) — категорически нехорошо и неправильно.

                  А вообще эта наука называется «аэрогидродинамика», это одна наука, если вам это надо объяснять — то, пожалуй, мне с вами дальше беседовать не о чем.

                  2all: самый бред начинается, когда высокий начальник на предложение, что бы снять все параметры, «пролить» на воде контур, предназначенный для неспешного течения фреона (или наоборот, продуть на воздухе контур для ж.натрия) — начинает вот точно так же втирать, что мол не надо путать аэро- и гидро- динамику… И этот его бред в итоге очень дорого обходится.


                  1. dobrobelko
                    25.08.2016 11:38

                    Я ождал подобной реакции. Вместо рекомендации поумерить снобизм просто предложу открыть курс аэродинамики MIT, где гидродинамика не упоминается. Пример самолета — MD-80. Дискуссию действительно лучше прекратить, с хамоватыми собеседниками я не общаюсь.


                    1. encyclopedist
                      25.08.2016 12:14

                      На западе разделяют fluid dynamics — это общая фундаментальная наука о течении разных сред (не важно, жидкостей или газов), aerodynamics — это прикладная наука об аэродинамике летательных аппаратов и прочих vehicle в воздухе.


                      1. dobrobelko
                        25.08.2016 12:15

                        Именно она имелась в виду.


  1. 5oclock
    24.08.2016 09:58
    +3

    «Существует история (я правда не знаю насколько она правдива), что одним из первых людей предложивших подобную теорию был никто иной как сам Альберт Эйнштейн. Согласно этой истории в 1916 году он написал соответствующую статью и на её основе предложил свою версию «идеального крыла»»
    Почему Эйнштейн? Вы же сами в статье упоминаете Жуковского.
    https://ru.m.wikipedia.org/wiki/Теорема_Жуковского
    У него в основе теоремы уже Бернулли заложен.


    1. 0serg
      24.08.2016 10:02
      +3

      Жуковский описывал правильный вариант — считал ток жидкости вокруг цилиндра и отображал его в ток жидкости вокруг крыла
      А Эйнштейн — неправильную версию основанную на разнице в длине пути над крылом и под ним


      1. 5oclock
        24.08.2016 10:26
        +4

        В любом случае — речь идёт о разнице скоростей воздушного потока.
        Т.е. в основе — всё тот же принцип Бернулли (если мы находимся в ламинарном потоке).


      1. SCareAngel
        24.08.2016 14:07

        В последней части, про преобразования из цилиндра и обратно, создается впечатление, что это это именно разоблачаемая подъемная сила.
        Если перевернуть крыло (то есть отразить преобразование цилиндра вокруг вертикальной оси) то направление подъемной силы сменится так же.
        Стоит так же заметить, что после преобразования, идеальная жидкость обтекает острую заднюю кромку крыла, и разделенный поток проходит одинаковый путь. И не создается никакой подъемной силы.
        А неидеальная жидкость не обтекает острую заднюю кромку, и разделенный поток, внезапно, проходит разное расстояние за одно время. И создается подъемная сила. И сила какая? Та же, что поднимает вращающийся цилиндр. А это именно та сила, что возникает в следствии закона Бернулли.


  1. dmitry_ch
    24.08.2016 10:04
    +13

    Напомнило:

    Почему самолеты летают


  1. ariksu
    24.08.2016 10:09
    +7

    Отличная статья. Хотелось бы отметить, что сила в потоке для вращающегося цилиндра носит название «Эффект Магнуса» и объясняет поведение кручёных мячей и парусов-роторов (тех, что в виде колонн).


    1. Alex_fisch
      24.08.2016 13:37

      А еще этот эффект нехило так влияет на дальность полета артиллерийского снаряд/пули из нарезного оружия в зависимости от направления ветра (слева или справа к траектории полета).
      Хотя в принципе это то же самое, что и полет крученого мяча.


    1. Ezhyg
      25.08.2016 06:48

      И подводных крыльев крутящихся, в виде брёвен цилиндров, есть «легенда», что именно от плавающего крутящегося бревна и придумано :).


  1. MaxxONE
    24.08.2016 10:26
    +3

    У вас первый же пример, где вы дуете на листок бумаги, — некорректен. Листок не взлетает вверх, а лишь поднимает свою заднюю часть, выравниваясь в потоке воздуха. Крыло же в аналогичной ситуации взлетело бы вверх, оставаясь параллельным полу (если опустить проблему устойчивости).


    1. 0serg
      24.08.2016 11:06

      Идея в том чтобы дуть не «вокруг» листка, а над его верхней поверхностью. Над листком воздух движется, под ним нет. В рамках наивного объяснения листок при этом поднимает бернуллевская разница давлений в движущемся и неподвижном воздухе.

      Интересно что в рамках подобного объяснения должно быть неважно как держать листок, вертикально или горизонтально, но на практике разница существует — «горизонтальный» лист загибается вниз под действием силы тяжести и «загибает» вниз поток воздуха проходящий над ним, формируя при этом подъемную силу.


  1. swelf
    24.08.2016 10:29
    +1

    >И хотя скорость потока над крылом действительно выше чем под ним, это является не причиной образования подъемной силы а следствием того что над крылом существует область пониженного давления, а под крылом — область повышенного

    Не совсем понял, это как, ведь давление создает именно воздух, и именно его потоки определенной конфигурации и скорости, а тут почему то их резделили.


  1. skyeff
    24.08.2016 10:29
    +4

    «Смешались в кучу люди, кони...»! Автору бы дописать текст до строчки «К сожалению, некоторые из подобных «простых объяснений» являются в корне неверными» и пойти спать, чтоб не плодить новых «простых объяснений».

    > Подъемная сила крыла зависит от его ориентации относительно набегающего потока воздуха — угла атаки

    Т.е. если поставить крыло под углом 85 градусов, самолет вертикально вверх рванет?!

    > Важный частный пример столь «не-бернуллевского» поведения наглядно демонстрируют экранопланы: при приближении крыла к земле его подъемная сила возрастает

    А где в уравнении Бернулли труба, или даже просто поверхность под крылом?

    Весь этот набор букв можно было уместить в пару фраз: "«Уравнение Бернулли» не единственная причина из-за которой летают самолеты. Существует множество других сил, влияющих на способность самолетов летать. Сложность описания совокупности действующих на крыло сил в реальных условиях такова, что в рамках научно-популярной статьи не имеет смысла. Интересующихся — добро пожаловать в авиамодельные кружки и институты.".


    1. 0serg
      24.08.2016 11:00
      +8

      Т.е. если поставить крыло под углом 85 градусов, самолет вертикально вверх рванет?!


      Прошу прощения, но в тексте написано просто «зависит от угла атаки» а не «непрерывно растет с увеличением угла атаки». Причем ниже по тексту этот момент подробно расписан — с графиками и объяснениями.

      А где в уравнении Бернулли труба, или даже просто поверхность под крылом?


      Хорошее замечание. Корректно было бы сказать что скорость должна вырасти у несжимаемого потока, что по закону Бернулли должно повлечь падение давления. Неявно здесь подразумевалась примерно вот такая картинка, иллюстрирующая принцип Бернулли:


      1. skyeff
        24.08.2016 11:16
        -9

        > Прошу прощения, но в тексте написано просто «зависит от угла атаки» а не «непрерывно растет с увеличением угла атаки». Причем ниже по тексту этот момент подробно расписан — с графиками и объяснениями.

        К сожалению, с самого начала в статье целый ряд довольно спорных утверждений. Увидев объем текста, читать дальше и выяснять, что же на самом деле хотел сказать автор, желания не возникло.


        1. PavelGatilov
          24.08.2016 18:24
          +2

          Но возникло желание пойти посрамить автора в комментариях?


          1. skyeff
            24.08.2016 18:45
            +2

            Нет, возникло желание указать на недостатки в изложении материала. Статья претендует на научно-популярную, о чем заявляется с самого начала. Более того, автор обещает развеять ряд «мифов», в число которых заносит и уравнение Бернулли. И начинает развенчания мифов с того, что расширяет границы модели за пределы тех, для которых формулировалось это уравнение. Зачем так делать, мне не понятно.

            На мой взгляд, правильнее было бы сказать, что формула Бернулли — это упрощенная модель, рассматривающая частный случай. А если взглянуть на происходящие процессы пристальнее, то вскрываются такие-то и такие-то противоречия, которые можно решить так-то и так-то.

            Тогда у людей будет формироваться образ преемственности научного знания. Если же подавать материал как он изложен, то возникает картина: была «неправильная» модель Бернулли, а вот вам новая «правильная» модель.


  1. 4ebriking
    24.08.2016 10:46

    натыкался ещё на такую картинку:

    (если съест теги — https://habrastorage.org/files/bbb/306/f85/bbb306f8511743418b2cb8ee0c231b42.jpg)


    1. 0serg
      24.08.2016 10:48
      +3

      Идея рисунка верная, а сам рисунок — нет: над крылом скорость потока все-таки выше :)


    1. grekmipt
      24.08.2016 11:34
      +3

      Вы возможно сильно удивитесь, но верны оба рисунка. )

      Просто в учебниках обычно пишут про крыло бесконечного размаха (длины). В этом случае мы имеем то что на верхней картинке. А если совсем точно — то скос потока вниз на первом рисунке тоже есть, но он бесконечно мал. Бесконечно малая скорость такого потока вниз умноженная на бесконечно большую массу потока (крыло бесконечное) создает конечный импульс, достаточный для удержания крыла. В этом и есть причина повышения аэродинамического качества крыла (отношения подъемной силы к лобовому сопротивлению) по мере удлинения крыла — чем длиннее крыло тем больше отбрасываемая им вниз масса воздуха в единицу времени, но тем на меньшей скорости нужно отбрасывать эту массу вниз для получения того же импульса вверх. Поскольку кинетическая энергия это квадрат скорости, а импульс — первая степень скорости, то получается энергетический выигрыш. И в пределе (крыло бесконечного размаха) требуется энергия (и скорость потока вниз) стремящаяся к нулю.

      Ну а на втором рисунке — тоже все верно, но там как раз крыло конечного размаха (реальное крыло) — поэтому скос потока вниз явный (а не бесконечно малый).


      1. 4ebriking
        24.08.2016 12:40
        +3

        Как я понял — основная суть рисунка в разрыве «неразрывной» струи, остальное вопрос масштабов изображения:
        image


        1. grekmipt
          24.08.2016 13:25

          Я понял рисунок по другому — на верхнем нет скоса потока вниз (на «выходе из крыла»), на нижнем — есть (а так же есть на имадже что Вы привели). Принципиальный вопрос который рассматривает автор статьи это причина возникновения подъемной силы крыла, и в самом общем виде — причина в том что крыло отбрасывает воздух вниз, все остальное уже «математические рюшечки». Соответственно, мой коммент и был в этом ключе — что скос потока есть в любом случае, но на верхнем рисунке он бесконечно мал по величине (в силу бесконечности размаха крыла).
          А разрыв потока это уже отдельный вопрос.


          1. encyclopedist
            24.08.2016 13:44

            Вы ошибаетесь. У любого крыла (и бесконечного и конечного) угол отклонения потока у кромки ненулевой. Нулевым он становится на большом отдалении от крыла.


            1. grekmipt
              24.08.2016 14:28

              Все верно, я говорю про ситуации удлинения крыла при сохранении его массы/площади. В этом случае по мере удлинения крыла будет требоваться все меньше энергии на его движение — при сохранении подъемной силы. Т.е. в пределе получим стремящийся к нулю скос потока и бесконечно длинное (ну и бесконечно узкое само собой) крыло.
              Для фиксированного по ширине крыла — Вы, разумеется, правы.


              1. Vladimir_Zaitsev
                24.08.2016 19:31

                И, в пределе, нулевая скорость потока?


                1. grekmipt
                  24.08.2016 21:33

                  Скорость потока вниз — да, в пределе нулевая. При массе потока стремящейся к бесконечности (что и дает требуемый конечный импульс).


  1. nkie
    24.08.2016 11:00
    +15

    Почему нельзя сказать по-простому, что при ненулевом угле атаки набегающий воздух уплотняется под крылом ( что и создает повышенное давление снизу), и соответственно разряжается над крылом (что и создает пониженное давление сверху). Разница этих давлений и создает подъемную силу. Нет никаких законов Бернули и скоростей, смысл по-моему тот же, что вы пытались донести


    1. 0serg
      24.08.2016 12:06
      +1

      Да, это верное и простое объяснение. Я пытался рассказать чуть более подробно почему он уплотняется под крылом и почему неверны альтернативные объяснения.


  1. YurySS
    24.08.2016 11:19
    +2

    Разница этих давлений формирует подъемную силу крыла и в соответствии с третьим законом Ньютона соответствующим образом отклоняет вниз часть набегающего воздушного потока — для того чтобы самолет мог лететь часть окружающего его воздуха должна непрерывно двигаться вниз.

    Интуиция мне подсказывает, что в этом случае возникает момент, стремящийся развернуть крыло в положение наименьшего сопротивления потоку воздуха. Чтобы предотвратить разворот крыла и превратить момент в подъёмную силу использует хвостовое оперение или иные способы.


    1. 4ebriking
      24.08.2016 11:32

      Там прикол в том, что разворачивает крыло — в положение НАИБОЛЬШЕГО сопротивления, но зато равного с обеих сторон относительно оси (вокруг которой разворачивает)


    1. 0serg
      24.08.2016 11:37
      +2

      Совершенно верно, это прекрасное замечание! Поправлю только что момент сил не обязательно разворачивает крыло в положение наименьшего сопротивления потоку. На практике это одна из причин по которым не существует некоего универсального профиля крыла которым бы пользовались все на свете. Разные профили создают разные моменты сил и по разному ведут себя при изменении угла атаки, изменении скорости потока и сваливании.


  1. qbertych
    24.08.2016 11:22
    +2

    Это чудесное объяснение ни разу не объясняет, почему самолет все-таки может летать вверх тормашками ;).


    1. grekmipt
      24.08.2016 11:36

      Так угол атаки же. Если крыло симметричное, то достаточно чтобы в положении кверх-тормашками хвост самолета был ниже носа (так чтобы у профиля крыла был штатный угол атаки). И отлично себе полетит ).


    1. vconst
      24.08.2016 11:41
      +4

      Объясняет. Профиль крыла для такого полета становится не оптимальным, но угол атаки все равно соблюдается


  1. Nameddin
    24.08.2016 11:40
    +1

    Из статьи так и не понял в итоге — как летают перевернутые самолёты-то? Ведь угол атаки получается отрицательный, верхняя поверхность крыла оказывается нижней и поток воздуха, получается, отклоняется вверх, а не вниз. Если профиль крыла симметричный, то понятно, что без разницы, а если не симметричный? Или они РУСом все время «подруливают» и так летят, читеры?


    1. 0serg
      24.08.2016 11:41
      +1

      Нос самолета в перевернутом полете просто направляют в сторону земли, так что крыло в таком полете перевернуто, но угол атаки у него положительный


      1. Odrin
        24.08.2016 12:45
        +5

        Все таки не в сторону земли, а наоборот. Независимо от перевернутости самолета, нос смотрит ввех. При этом для удержания высоты в перевернутом состоянии кабрирование должно быть больше, чем в нормальном состоянии.


        1. 0serg
          24.08.2016 12:46

          Да, верно, спасибо. Я запутался, поздно сообразил и уже не мог исправить комментарий.


      1. igruh
        24.08.2016 13:07

        Хвост. /Забыл обновить комментарии.../


      1. Andy_Big
        24.08.2016 13:35

        del


  1. scvorec3
    24.08.2016 11:41
    +1

    Лично мне в этой статье не хватает схемы/рисунка где показывается, все-таки, почему возникает подъемная сила, если самолет летит вверх тормашками


    1. 0serg
      24.08.2016 11:42

      1. vconst
        24.08.2016 11:46

        Лучше добавьте ее в текст статьи


        1. 0serg
          24.08.2016 11:48
          +1

          Добавил


  1. gsaw
    24.08.2016 12:09
    +1

    В детстве сидел у иллюминатора возле крыла. При входе в облака крыло начало трясти, я подумал тогда, что самолет крыльями машет и потому летит. Вы меня разочаровали.


  1. ivan01
    24.08.2016 12:12
    +3

    Еще одно «простое объяснение». Метод Жуковского для расчета подъемной силы через циркуляцию не является объяснением. Тем более, что наиболее корректное объяснение эффекта Магнуса лежит опять таки через уравнение Бернули, совсем не понятно, чем оно вам так не понравилось! Объясните ка почему вращающийся цилиндр в потоке имеет подъемную силу!
    А корректное, реальное объяснение должно лежать через баланс импульса, на рассуждениях о работе сил вязкости и образующихся полях давления.
    Ваши попытки поставить давление на первое место просто смешны. Потому как это давление создается движением крыла, а не наоборот. Работа сил вязкости приводит к изменению поля скорости, и эта работа компенсируется изменением давления. Которое уже приводит к образованию подъемной силы.


    1. 0serg
      24.08.2016 12:27
      -1

      Уравнение Бернулли само по себе является совершенно правильным способом пересчитать поле скоростей несжимаемого потенциального потока в давление. Вот только само это поле скоростей рассчитать проблематично. Вращающийся цилиндр в этом плане является приятным исключением.

      Насчет работы сил вязкости мне кажется Вы неправы. От самой формулировки о работе сил вязкости до наблюдения о том что подъемная сила крыла существует и в потоке со сколь угодно малой (главное чтобы не нулевой) вязкостью. В частности иногда использующийся тезис о том что линии тока искривляются именно за счет вязкости является ошибочным. Вязкость участвует только в формировании областей давления вокруг крыла, но дальше они устойчивы сами по себе.


      1. ivan01
        24.08.2016 12:37
        +2

        С чего бы это? Разве вязкость вдали от крыла исчезает?
        Без вязкости невозможен отрыв погранслоя. А в безотрывном течении вокруг тела вдали от других тел в случае отсутствия вязкости действует парадокс Д'Аламбера и никакое сопротивление или подъемная сила невозможны. Следовательно она, вязкость все таки.
        А так я согласен, все легко объясняется через уравнение Бернули. Не понятно почему автор его отбросил в самом начале, а потом снова косвенно притянул за уши в методе Жуковского.


        1. encyclopedist
          24.08.2016 12:58

          <удалено>


        1. 0serg
          24.08.2016 13:36

          Давайте рассмотрим цилиндр помещенный в неподвижную идеальную жидкость. Можно показать что существует решение уравнений Эйлера для несжимаемой идеальной жидкости в котором поток жидкости равномерно вращается вокруг цилиндра, быстро у его поверхности и все медленнее и медленнее — по мере удаления от неё.

          Теперь возьмем суперпозицию решения Д'Аламбера и указанного специального решения с вращающимся потоком. Это решение тоже будет удовлетворять уравнениям Эйлера и описывать движение несжимаемой жидкости вокруг цилиндра при отсутствии вязкости.

          Согласно парадоксу Д'Аламбера, полученное гибридное решение не может давать силу сопротивления. Но это верно только для силы сопротивления, т.е. проекции суммарной силы действующей на цилиндр на направление в котором движется жидкость на бесконечности. А вот перпендикулярно этому направлению сила возможна и можно показать что в подобном гибридном решении она, собственно, и возникает.

          Вязкость же здесь возникает в роли стабилизирующего фактора. Решение уравнений Эйлера нестабильно — любое возмущение в идеальном решении экспоненциально растет и приводит к режиму хаотичного турбулентного обтекания цилиндра. Наличие даже небольшой вязкости с одной стороны гасит эти колебания, а с другой — смещает у крыла точку равновесия к которой стремится решение именно к этому конкретному решению с циркуляцией.


          1. Bronx
            27.08.2016 01:58

            > существует решение уравнений Эйлера для несжимаемой идеальной жидкости в котором поток жидкости равномерно вращается вокруг цилиндра, быстро у его поверхности и все медленнее и медленнее — по мере удаления от неё.

            Что заставит идеальную жидкость без вязкости вращаться вокруг цилиндра?


            1. 0serg
              27.08.2016 09:12

              Вокруг цилиндра — ничего. Но мы отображаем конформным отображением решение для цилиндра на случай крыла и из набора возможных решений для цилиндра получаем набор возможных решений для крыла

              А дальше действуют два фактора:
              1. То что во всех решениях кроме двух (одного если считать поток встречным) жидкость должна огибать острую заднюю кромку крыла
              2. Неустойчивость решения с нулевой вязкостью (и устойчивость к малым возмущениям при не-нулевой)

              А дальше делаются следующие предположения:
              1. Если добавить жидкости немного вязкости то решения будут почти такими же
              2. Но при сколь угодно малой вязкости огибание потоком задней кромки невозможно

              Отсюда выдвигается постулат что единственным устойчивым решением обтекания крыла потоком идеальной жидкости с малой, но отличной от нуля вязкостью при отсутствии турбулентности будет решение которое почти не отличается от того единственного решения для невязкой идеальной жидкости которое дает отрыв линий тока на задней кромке крыла.

              В принципе отсюда не следует что система из какого-то начального положения обязательно сойдется именно к такому решению (и срыв потока — наглядный тому пример), но если сойдется — то именно к этому. В экспериментах это довольно интересно выглядит — с задней кромки крыла при резком начале движения крыла отрывается небольшой вихрь после чего поток течет устойчиво


              1. Bronx
                27.08.2016 11:20

                Вы ранее написали, что «тезис о том что линии тока искривляются именно за счет вязкости является ошибочным.»

                Однако, насколько я понимаю, без вязкости не будет и циркуляции, а без неё не будет и решения со скосом потока и, соответственно, подъёмной силой (даже не важно, с пограничным слоем это решение, хоть без оного). Поэтому мне кажется неправильным сводить роль вязкости исключительно лишь к стабилизации решения.


                1. 0serg
                  27.08.2016 21:32

                  Так решение со скосом потока и циркуляцией как раз существует независимо от наличия вязкости. Да, оно а) неустойчиво и б) непонятно почему из всех возможных решений будет реализовано именно оно. Но оно существует и линии тока там искривляются при полностью нулевой вязкости жидкости.


                  1. Bronx
                    27.08.2016 23:08

                    Решение есть, если предположить, что присоединённый вихрь УЖЕ существует, как начальное условие. Скажем, где-то уже есть система вихрей, и мы моментально засунули в него крыло.

                    Но в реальности начальным условием является невихревой потенциальный поток. Чтобы прийти от системы с потенциальным потоком к системе с присоединённым вихрем, нужна будет вязкость, иначе опять получается парадокс д'Аламбера. Что-то должно породить и поддерживать присоединённый вихрь — и тут роль вязкости ключевая, имхо.


                    1. 0serg
                      28.08.2016 10:04

                      Поток с циркуляцией вокруг крыла является потенциальным. Это несколько неочевидно, но тем не менее. На его поддержание не требуется работы.


                      1. Bronx
                        28.08.2016 12:50

                        Хм, и впрямь. Пожалуй, вы меня убедили, спасибо.


  1. FransuaMaryDelone
    24.08.2016 12:18
    +12

    Дело в том что...

    Дело в том, что те, кому надо, те каждый раз решают или Навье-Стокса или Больцмана. А статья похожа на цыганский табор — много шума, много бестолковой экспрессии и неприятное ощущение, что сейчас кого-нибудь обманут.


    1. 0serg
      24.08.2016 12:32
      -2

      Мне и, полагаю, многим другим людям хочется все же иметь разумно верное приближенное объяснение того как возникает подъемная сила и летает самолет, а предложение «решить уравнения движения потока вокруг крыла» эту задачу, к сожалению, не решают. К тому же уравнения Навье-Стокса или Больцмана — это, конечно, хорошо, но часто полезно иметь и простую модель из которой можно получить приближенные или качественные оценки для проверки правильности более точного решения.


      1. FransuaMaryDelone
        24.08.2016 13:46
        +4

        > «решить уравнения движения потока вокруг крыла» эту задачу, к сожалению, не решают.

        Еще как решают. Решали и будут решать.

        > часто полезно иметь и простую модель из которой можно получить приближенные или качественные оценки для проверки правильности более точного решения

        Что?! Для проверки правильности?! Часто полезно?! Это неправда, Вы не сможете доказать, что Ваша умозрительная пальцевая модель адекватна реальности более, чем уравнения Навье-Стокса.

        > Мне и, полагаю, многим другим людям хочется все же иметь разумно верное приближенное объяснение

        Знаете, тихо иметь свое мнение не вредно. Но прилюдное сенсационное опровержение «мифов» создает у не владеющих темой людей обманчивое чувство владения, что сказывается потом.


  1. marenkov
    24.08.2016 12:30

    «Классическая» теория подъемной силы напоминает историю с голым королем. Вроде бы разум и тот факт, что плоский кусок фанеры довольно не плохо летает, говорят о том, что профиль не главное, но ведь все вокруг утверждают, что все дело в профиле и разнице скоростей потока, а тот кто не согласен с этим ничего не понимает в аэродинамике.


    1. ivan01
      24.08.2016 12:39

      Вы наверное их не так понимаете. Все дело в профиле, когда мы говорим об эффективности. А фанера это такое херовое, но крыло, с этим никто не спорит.


      1. marenkov
        24.08.2016 12:50

        Согласно теории о разнице скоростей потоков воздуха у плоской фанеры должна быть нулевая подъемная сила. Однако это не так. Напрашивается вывод, что фокус все же в отклонении потока вниз.


        1. ivan01
          24.08.2016 12:58

          Это сфига ли? Снизу поток об фанеру тормозится, сверху нет. Разница есть. Дело в торможении и в ускорении.


          1. marenkov
            24.08.2016 13:17

            Да-да, именно так, из-за торможения воздуха под крылом получаем зону высокого давления, над крылом образуется «дыра» — зона низкого давления. А разница в длине пути, которую проходит воздух тут ни при чем. В целом тут история с курицей и яйцом — то ли высокое давление потому, что скорость потока ниже, толи скорость потока ниже потому, что он встретил зону высокого давления.


            1. ivan01
              24.08.2016 13:22

              При чем здесь этот путь? Просто выкиньте его из объяснения и более лучшего объяснения на пальцах вы не придумаете. А чтобы объянить почему он там затормозился и вывести интегралл Бернули так это надо курс лекций читать.


              1. marenkov
                24.08.2016 13:33

                В итоге приходим к тому, что подъемная сила — результат разницы давлений под и над крылом. Благодаря чему возникает эта разница — это уже другой вопрос, и тут возможны варианты и их комбинации: разница скоростей потока, отклонение потока вниз, восходящие потоки воздуха…

                Кстати, в универе на основах авиастроения учили, что профиль лишь обеспечивает ламинарность потока и отсутствие срыва на максимальных углах атаки.


                1. ivan01
                  24.08.2016 14:44

                  Правильно учили. Апгрейд плоской пластины.image для предотвращения срыва вообще жертвуют частью давления под крылом и создают каналы в крыле.


        1. ivan01
          24.08.2016 13:02

          ~~~~~~~~~~~~~~~~~~~~~~~~~~~~~ — вязкость внешнего потока (типа стенка)

          ДИФФУЗОР
          .._
          … \_
          .......\_
          ..........\_ — фанера под положительным углом атаки
          .............\_
          ................\_

          КОНФУЗОР

          ~~~~~~~~~~~~~~~~~~~~~~~~~ — вязкость внешнего потока (типа стенка)

          Хотите объяснения на пальцах, пжалста, отрывом пренебрегли, нет его пока.


          1. ivan01
            24.08.2016 13:07
            +1

            Нет, фигня какая-то выше вышла )


    1. AleXP3
      24.08.2016 14:06

      Вообще-то не правильно говорить, что «все вокруг утверждают, что всё дело в профиле и разнице скоростей потока...». В авиастроении есть масса разных профилей крыла, в т.ч. и много «симметричных».

      http://kipla.kai.ru/liter/Spravochnic_avia_profiley.pdf — очень интересный документ.


    1. LynXzp
      25.08.2016 17:20

      Да, правильно, еще со школы помню свои рассуждения по этому поводу: 1 самолет может лететь вверх ногами, 2 самолет может лететь с плоским крылом, даже без трения, если крыло «толкает» воздух вниз то само будет отталкиваться вверх. Когда говорил одноклассникам — это вызывало у кого-то боль. :) (Ест-но речь только о самолетах с двигателем)

      Ну а остальное это уже допиливание напильником.


  1. ssneg
    24.08.2016 12:35
    -4

    image


  1. koshi
    24.08.2016 12:47
    +1

    Ключевые вопросы, мне кажется, здесь следующие:
    1) есть ли эффективная математическая модель явления (например, в виде системы дифференциальных уравнений)?
    2) если такая модель имеется, насколько она обоснованая физически (через законы Ньютона, Навье-Стокса или чего-то подобного)?

    К сожалению, автор не даёт на них ответов.

    Кстати, есть интересное альтернативное описание проблемы подъёмной силы крыла в историческом аспекте:
    https://kniganews.org/map/e/01-01/hex54/
    Тоже, правда, без ответов на ключевые вопросы.


  1. CyberTIM
    24.08.2016 14:06

    Помню, в институте на дисциплине аэродинамики профессор показывал картинку разрыва потока, увеличения скорости сверху, снижения снизу и объяснял всё это законом Бернулли. Однако, я никак не мог понять какое отношение Бернулли имеет к газам? Все его постулаты о механики жидкостей. В жидкостях главное её неразрывность, чего о газах не скажешь.


    1. encyclopedist
      24.08.2016 14:16

      У жидкостей и газов одинаковая механика (пока давления газа достаточно большие).


    1. ivan01
      24.08.2016 14:45

      Газ можно считать несжимаемым, пока скорости не очень большие. Неразрывность есть и там и там.


  1. xenohunter
    24.08.2016 14:12

    Спасибо за статью! Считаю, что ко всем научно-популярным источникам нужно относиться с интересом, но осторожно. Буквально недавно наткнулся на эту самую ложную гипотезу о подъёмной силе крыла, и сразу закрались подозрения. Однако, физики пока не знаю, и времени разбираться не было, поэтому просто отложил эту идею в сторону как непроверенную. Кстати, на последнем изображении справа разве не должна быть большая плотность жидкости под цилиндром, а не над ним?


    1. encyclopedist
      24.08.2016 14:19

      Плотность (частота) линий тока показывает не плотность среды, а её скорость.


      1. xenohunter
        24.08.2016 14:20

        Тогда ясно, почему так, спасибо!


    1. 0serg
      24.08.2016 14:20

      Плотность во всех выкладках предполагается константой. Распределение давлений вокруг цилиндра (и вообще более подробные и полные выкладки с расчетами этого случая) можно посмотреть вот здесь:

      http://www-mdp.eng.cam.ac.uk/web/library/enginfo/aerothermal_dvd_only/aero/fprops/poten/node40.html


  1. Bedal
    24.08.2016 14:41

    Автор не справился со своими знаниями, которые у него, бесспорно есть. Результат не ахти. Извините.
    Можно разобрать подробнее, но придётся в том числе почти все комменты цитировать :-)


  1. chaynick
    24.08.2016 14:42

    «Если в Вашем объяснении фигурируют «разная длина верхней и нижней поверхности крыла», «разная скорость потока воздуха на верхней и нижней кромках крыла» и «закон Бернулли», то я вынужден Вам сообщить, что Вы скорее всего стали жертвой популярнейшего мифа, который преподают порою даже в школьной программе.»

    Суть статьи состоит в пересказе теоремы Жуковского, но почему-то с указанием того, что закон Бернулли неприменим к крылу. Самое забавное, что теорему-то можно вывести из закона Бернулли.

    Собственно уравнение Бернулли это закон сохранения энергии в неком объеме жидкости/газа, оспаривать его ой как глупо. ИМХО из принципа Бернулли следует что отношение давления/скорости объема/плотности объема постоянно во времени, так что говорить что из-за разной скорости потоков возникает подъемная сила так же грамотно как и то что подъемная сила возникает из-за разницы давлений над/под крылом, так как все это НЕРАЗРЫВНО связано. Однако, на закритических углах атаки крыло по-прежнему отклоняет поток вниз но почему-то подъемная сила драматически уменьшается, так что подъемная сила — это НЕ результат исключительно отклонения потока воздуха вниз.

    Но самая лютая ошибка — утверждать что эффект экрана не подчиняется закону Бернулли. Закон сохранения энергии выполняется всегда, у вас просто неверная интерпритация закона. Эффект экрана возможен при 1) наличии экрана площадью много больше крыла 2) крыла конечного размера, и работает он за счет уменьшения индуцированных концевых вихрей и переотражения воздушных волн от поверхности экрана. Не верите — попробуйте описать работу экранного эффекта самостоятельно.


    1. ivan01
      24.08.2016 14:49
      +1

      Срыв погранслоя так плохо влияет потому что уравнение Бернули действует вдоль линии тока, линия тока оторвалась от крыла — всё.


    1. ivan01
      24.08.2016 14:52

      Не совсем так, я бы назвал это все таки воздушной подушкой между крылом и землей. Просто условия возникновения этой подушки описывают через время прохождения возмущения. т.е. звуковой волны. Время прохождения звука от крыла к земле и обратно должно быть меньше времени пролета крыла, поэтому они такие широкие.


      1. chaynick
        24.08.2016 15:03

        Переотражение это по сути и есть воздушная подушка. Фронт волны отразился от экрана и вернулся назад. Следствия — проблемы с управляемостью экраноплана из-за разницы точек приложения подъемной силы и эффекта экрана и принципиальная невозможность существования сверхзвуковых экранопланов. Но также близость экрана разбивает концевые вихри, что положительно сказывается на аэродинамическом сопротивлении.


        1. ivan01
          24.08.2016 15:24

          Неа, это скорость распространения возмущений в среде, вот что важно, пока звуковая волна не долетела до земли, воздуху там глубоко наплевать что там сверху пролетает экраноплан, волшебный пони или бабочка. Подушка возникает не из-за того что волны какие-то летают. Просто с их скоростью по системе распространяется информация.


          1. chaynick
            24.08.2016 16:14

            О_о Подушка это наложение фронтов воздушных волн от крыла. Информационное воздействие — это к торсионщикам.


            1. ivan01
              24.08.2016 16:47

              Почему это? Информация об изменении в какой-то части системы распространяется со скоростью звука (если мы не рассматриваем детонационные волны). Считайте, что я численно описываю.
              Наложение фронтов? Нет, не согласен, никакого фронта от земли при пролетании экраноплана не распространяется, фронт только отражается от нее, нет никакого наложения. Также нет никакой и волны, волна здесь скорее виртуальная, в виде распространения возмущения по среде. Если вы сожмете поршень в километровом шприце, то у иголки давление поднимется только через 3 секунды, но в шприце давление поднимается не из-за распространения волны, а из-за изменения объема. Так же и тут подушка появляется из-за движения крыла, необходимое условие, чтобы она была под крылом как раз вытекает из скорости звука.


              1. chaynick
                24.08.2016 17:59

                http://ss-op.ru/files/photos/ekrplan/effect_ekrana.gif

                Фронт волны это как раз попытка воздуха выравнять свою плотность из области высокого давления под крылом. И так как область эта перемещается вместе с крылом, то такие фронты порождаются постоянно.


                1. ivan01
                  24.08.2016 19:08

                  Какой волны? Звуковой? Ну замечательно, теперь звуковыми волнами будем аэродинамику объяснять?
                  Что это за белая линия на картинке? Лучик звука?! Нет, Линия тока? Ой тоже нет. А что же тогда?
                  Я просто повторю, что вы написали: «Фронт волны это как раз попытка воздуха выравнять свою плотность из области высокого давления под крылом.» Значит создана эта область не волной, так? Или она создана волной и волна там что-то пытается?
                  Поясните этот бред.


                  1. chaynick
                    25.08.2016 13:03

                    Если совсем грубо — молекула газа. Звук — это (вы не поверите!) как раз прекрасно описывается аэродинамикой… Или скачок уплотнения у вас перестал быть звуковой волной? Область высокого давления создается набегающим потоком воздуха в соответствии с законом Бернулли под крылом. Ударная волна вам в пример, если не понимаете что такое фронт волны при выравнивании давления. И сразу обратите внимание на то, как деформируется волны под летящим экранопланом.

                    https://www.youtube.com/watch?v=vHV8fNffJqQ
                    http://ss-op.ru/files/photos/ekrplan/orlenok/orlionok_03.jpg


                    1. ivan01
                      25.08.2016 13:39

                      Вы реально верите, что это путь молекулы газа? Звук описывается да, теми же уравнениями навье стокса, но в звуковой волне нет скачка, параметры после прохождения звуковой волны практически не меняются. Более того звук распространяется со скоростью звука, а ударные волны быстрее. Поэтому вопрос был ли скачек уплотнения звуковой волной… он какбы не стоит, скачек уплотнения разделяет области с разными параметрами газа, при его прохождении параметры меняются посчитать их можно по уравнению Гюгонио и ударным полярам для данного газа. Ударной волны при движении экраноплана нет, еще раз повторяю, просто потому что ей взяться неоткуда. Все возмущения распространяются со скоростью звука. Это сраный дозвук!


  1. Tim_23
    24.08.2016 16:30

    Объяснение Ньютона кстати не лишено смысла, так как при развороте потока действительно передаётся импульс, и сила действующая на пластину больше чем лобовое сопротивление( из закона сохранения импульса), но при этом крыло должно иметь бесконечную длину и поток должен полностью разворачиваться.

    Кстати, аналогии работают и в газодинамике. Есть американский стандарт НАСА, в котором сила набегающего потока струи на газотражатель насчитывается с учётом полного разворота его и получается, что сила на газотражатель больше чем тяга ракеты. Вот такие сюрпризы. Опять же эта теория не учитывает скачки уплотнения, растеканием газов и прочее.


  1. leonf08
    24.08.2016 16:55

    Я совсем не специалист в аэродинамике. Тоже всегда думал, что самолеты летают благодаря особой форме крыла. А теперь ничего не понимаю. Единственное, что понял, так это то, что есть лишь теории и попытки объяснить. То есть выходит, что инженеры, энтузиасты в начале ХХ века создали самолет и он летает, а почему летают только пытаются объяснить. Поправьте меня, если я неправ.


    1. encyclopedist
      24.08.2016 17:28
      +1

      Инженеры и ученые знают почему оно летает. Ответ звучит как "так следует из системы уравнений Навье-Стокса". Проблемы возникают когда пытаются объяснить это "на пальцах".


  1. vvzvlad
    24.08.2016 17:46
    +1

    А поскольку вращающийся цилиндр в потоке жидкости создает подъемную силу, то ее создает и соответствующее крыло.

    Это вот случайно не этот эффект?


    1. 0serg
      24.08.2016 17:53

      Он самый



    1. BACRHR
      01.09.2016 18:45

      еще и в футболе https://www.youtube.com/watch?v=NWkuOY-XNt8


  1. Dum_spiro_spero
    25.08.2016 00:01

    Насчет парадокса Даламбера — не все так просто.
    Любопытная статья как-то попалась — сейчас нашел её, привожу ссылку и пару выводов — возможно покажется интересным.
    — Применение потенциальной безвихревой модели для решения задач установившегося стационарного обтекания идеальной средой твердых тел обязательно приводит к решениям с нулевыми силовыми взаимодействиями (в соответствии с законом сохранения и превращения энергии).
    O Поэтому в парадоксе Д’Аламбера нулевая сила сопротивления движению твердых тел в идеальной среде не является результатом теоретических расчетов – нулевой конечный результат уже заложен при бессистемном произвольном выборе начальных условий (до начала вычислений), т.е. при выборе безвихревой модели обтекания.
    — http://dalamberparadox.narod.ru/


    1. ivan01
      25.08.2016 11:37

      Что они этим хотят сказать? Это понятно из определения.


  1. Oroblanco
    25.08.2016 00:29

    Автор абсолютно прав, так и есть. Угол атаки и скорость важнее, чем профиль. И тут особо показательны не самолёты, а боевые ракеты (воздух-воздух, например). У них симметричный профиль крыла, нет «верха» и «низа» в горизонтальном полёте. Ракета иногда ещё вращается в полёте, а самые примитивные варианты ракетных крыльев — штамповка из листа металла (тот самый «лист фанеры»). Причём бОльшую часть подъемной силы ракете даёт корпус, который на скорости работает в качестве крыла. Но с точки зрения классической аэродинамики это «никакое» крыло, поэтому боевые ракеты имеют дополнительные плоскости, которые упорядочивают силы. Так что классический профиль крыла — это для медленного и печального полёта в ламинарных потоках. А в реальности может лететь кусок трубы, если достаточно энергии для скорости и есть плоскости для управления и стабилизации.


  1. Imbecile
    25.08.2016 07:20

    Никогда не думал, что буду вспоминать аэрогидродинамику после универа.
    1. Несжимаемая жидкость. Несжимаемый воздух. Режет глаз. При прочих равных условиях, разница между газом и жидкостью именно в том, что первый сжимается, а вторая — нет. Как минимум в классической модели с уравнениями Эйлера.
    2. Странно, что при рассмотрении экранного эффекта не упомянули обязательную конусообразную ударную волну чуть впереди крыла и её отражение от поверхности в направлении крыла.


    1. 0serg
      25.08.2016 08:25

      1. Это верно, но чтобы сжать воздух к нему нужно приложить усилие. Поскольку атмосферный воздух уже сжат атмосферным давлением это усилие может быть очень большим — скажем чтобы сжать воздух вдвое у поверхности земли нужно усилие в 10 тонн на м2 поверхности соприкасающейся со сжимаемым воздухом. А в случае крыла мы даже не можем обжать воздух со всех сторон — если мы давим на него снизу то вместо того чтобы сжиматься он может просто уйти вверх. На практике в итоге для большинства случаев поток воздуха вокруг крыла ведет себя почти как несжимаемая жидкость — усилия которые на него действуют слишком малы чтобы его заметно сжать

      2. Я не рассматриваю природу возникновения экранного эффекта и не понимаю почему это так многих зацепило. Я говорю только что это удобный пример в котором наивная попытка угадать поле скоростей воздуха под крылом и вывести из этого давление дает результат прямо противоположный действительности.


    1. ivan01
      25.08.2016 11:45

      1. Ну ладно, есть у нас поток 50 м/с допустим, мы его затормозили, давление торможения по уравнению Бернули равно 1кг/м^3*50м/с*50м/с/2=1250 Па, атмосферное давление 10^5 Па, ок, давление выросло на 1.25%, это приведет к совершенно незначительному изменению плотности. Считаем что газ несжимаем.
      2.Какая еще ударная волна в дозвуковом обтекании?!


      1. Imbecile
        25.08.2016 12:07

        1. Вопрос в том, что термины газовой динамики и реальной жизни — различаются. Если среда не сжимается, то она считается жидкостью. Не важно, какое у неё реальное состояние. Если модель подразумевает, что нельзя пренебрегать изменением плотности, то среда считается газом.
        2. Обычная ударная волна, которая формируется перед крылом. Крыло вызывает малые возмущения плотности. Эти возмущения распространяются со скоростью звука (по сути, скорость звука — это и есть скорость распространения малых возмущений). Каждое «новое» возмущение идёт по более плотной среде. Следовательно, распространяется быстрее, чем предыдущее. По мере распространения в трёхмерном пространстве, малое возмущение затухает. Но сзади его «подпитывают» новые возмущения. В итоге мы получаем, что перед крылом движется фронт резкого перепада давления, который и называется ударной волной.


        1. ivan01
          25.08.2016 12:47

          Мда, а ник то у вас говорящий. Более дикой фантазии на тему ударных волн я не слышал. Что такое ударная волна? Это скачкообразное изменение какого-либо параметра среды, в частности плотности и давления. При дозвуковом движении крыла никаким скачкообразным изменениям взяться просто неоткуда, разве это не очевидно? Нет никаких ударных волн на дозвуке перед крылом.

          Крыло вызывает малые возмущения плотности. Эти возмущения распространяются со скоростью звука (по сути, скорость звука — это и есть скорость распространения малых возмущений). Каждое «новое» возмущение идёт по более плотной среде.
          Какие такие возмущения? Шум турбулентного погранслоя? Шум от вибрации двигателя? И что мешает ему уходить вдаль от крыла? Сверхзвукового потока против которого он не может распространяться нет.
          Каждое «новое» возмущение идёт по более плотной среде
          Вот этот бред? Это без коментариев, это просто какая-то фантазия. Как можно опровергнуть фантазию я хз.


          1. Imbecile
            25.08.2016 13:47

            Волна сжатия
            — распространение бесконечно малого или конечного возмущения Давления (?)p < 0 в покоящейся или движущейся среде. В идеальном газе малое возмущение распространяется со скоростью звука. Для непрерывной последовательности бесконечно малых возмущении каждое последующее возмущение распространяется в среде с большей скоростью из-за повышения температуры, постепенно догоняет предыдущее и сливается с ним. В результате этого процесса слияния образуется ударная волна.


            1. ivan01
              25.08.2016 13:54

              Хаааа, я понял, ну это вы заблуждаетесь. Такого никогда не произойдет, а знаете почему? Потому что звук затухает. Он же нагревает среду, так? Энергия уходит и пффф, такого нет в реальности. Это интересная особенность мат. модели.


            1. ivan01
              25.08.2016 13:57

              Совет вам, читайте книжку целиком, тогда придет понимание, гуглить куски в надежде что вам это поможет в споре это плохой путь. Источник то хороший может быть, а вот пользоваться им надо уметь.


  1. loverussia
    25.08.2016 12:31

    Все бы хорошо… Но вы почему-то не упомянули, о том что когда мы пытаемся подсчитать точно подъемную силу при конкретных размерах крыла, то все наши расчеты математические не совпадают с результатом измерений практических…
    И ученые считают, что на самом деле в образовании подъемной силы учавствуют квантовые эффекты… Или вы об этом даже не знали? )))


    1. ivan01
      25.08.2016 12:48

      Кто это так считает? Фамилию в студию! Оч интересно!


      1. loverussia
        25.08.2016 14:48
        -1

        https://kniganews.org/map/e/01-01/hex54
        Вот здесь цикл очень интересных статей…


    1. Dum_spiro_spero
      27.08.2016 02:13

      Э… я ученый. Да. Хотя Л.Д. Ландау считал, что это ослы бывают учеными — а мы — научные работники. Ну да ладно. Не буду ёрничать. Дело в чем — посчитать численно обтекание крыла непросто. Крылом я не занимался, возился с цилиндром. Нюансов масса, начиная от мерности задачи — начиная с какого-то Re становятся существенными трехмерные эффекты, продолжая банальным количеством ячеек, и дальше уже все прелести сверхзвука, и т.п. Проблемы эти смешанные — физико-математические. Но качественных проблем на сегодня нет — т.е. берем ячеек побольше, порядок схем повыше, компьютер помощнее — и получим очень хорошее совпадение с экспериментом по всему.


  1. Mountlander
    25.08.2016 12:31

    Как я себе представляю. Речь принципиально о том, что не давление падает в результате повышения скорости воздуха над крылом, а наоборот скорость потока растет в области низкого давления.
    А как же тогда школьный опыт — берем длинную часть от авторучки и дуем вверх. На нее опускаем теннисный шарик.
    Он никуда не падает, а держится в воздухе. Если чуть изменить направление воздушной струи, то шарик снова в нее войдет. Мне кажется, что объяснение, которое было раньше больше подходит для этого опыта. Т.е. Где скорость потока воздуха выше, там меньше давление, а вокруг давление выше и шарик сам встает в струю, а не наоборот — мы уменьшаем давление в потоке воздуха, и скорость там растет… имхо…


    1. 0serg
      25.08.2016 13:31

      Общая проблема рассуждений о том что в движущемся с большой скоростью потоке будет низкое статическое давление состоит в том что принцип Бернулли применим только к движению вдоль одной конкретной линии тока воздуха в отсутствии внешнего источника энергии. Если две линии начинаются рядом в области одинакового давления, а затем одна из них разгоняется то в ней давление станет ниже чем во второй линии. Но если две линии начинаются из областей с разным давлением то это вовсе не обязательно будет так. И у фена статическое давление в струе как правило больше атмосферного.

      Поток, давление в середине которого меньше атмосферного будет засасывать в себя не только шарики, но и окружающий воздух, а это очень быстро его тормозит и делает неустойчивым. Это целая проблема для создателей реактивных и ракетных двигателей поскольку лишнее давление в выходном потоке газа означает потерянную впустую энергию. Создатели фенов обычно просто создают струю воздуха высокого давления.

      Шарик удерживает в струе воздуха не столько то что там «низкое давление» (это может быть действительно так, но как правило нет), а то что за шариком есть своеобразная аэродинамическая «тень» давление в которой ниже чем в остальной части потока. Эта тень заставляет поток отклоняться в сторону шарика а в соответствии с третьим законом Ньютона если шарик действует на поток силой которая отклоняет его в свою сторону, то поток должен действовать на шарик силой которая будет тянуть его в противоположном направлении (к центру потока). В терминах линий тока, шарик изгибает эти линии вокруг себя в силу чего в направлении перпендикулярном этим линиям будет существовать градиент давлений по направлению от шарика.


  1. ivan_petroff
    25.08.2016 12:31

    Очень запутанное объяснение о принципах полета самолета, где основным конструктивным элементом за счет которого держится самолет в воздухе является крыло.

    Все пассажирские самолеты, а также беспилотные самолеты(которыми видимо занимается автор) в настоящее время дозвуковые и сжатием воздуха можно пренебречь. Любой самолет держится в воздухе за счет подъемной силы крыла, которая создается воздушным потоком, в результате поступательного движения самолета за счет силы тяги самолета.

    Подъёмная сила — составляющая полной аэродинамической силы, перпендикулярная вектору скорости движения тела в воздухе, возникающая в результате несимметричности обтекания тела потоком.

    Крыло на дозвуковых скоростях имеет имеет поперечный профиль, где верхняя поверхность имеет более выпуклую поверхность, чем нижняя. За счет такого профиля создается разность статического давления между нижней и верхней поверхностью, которая собственно держит самолет в воздухе в диапазоне скоростей от минимальной до максимальной и хорошо описывается в соответствии законом Бернулли. Статическое давление среды в тех областях, где скорость потока более высока, будет ниже, и наоборот.

    Экранный эффект вблизи поверхности (водной или земной) также хорошо описывается в соответствии с законом Бернулли. На малой высоте происходит торможения потока между нижней поверхностью профиля крыла и поверхностью экрана. При торможении потока возрастает статическое давление при приближении профиля к экрану и, следовательно, подъемная сила увеличивается, а при удалении от экрана — уменьшается.

    Теоретически можно только оценить расчетные характеристики самолета приближенно, а уточнение модели производится на моделях и в натурных экспериментах. Для этого строятся поляры самолета на моделях в аэродинамической трубе, а затем уточняется при испытании самолета.

    Приведенные примеры о полетах самолета в перевернутом виде к объяснению обтекания крыла не имеют никакого отношения. Самолеты для пилотажа специально проектируются и имеют ряд конструктивных отличий от пассажирских, последние ограничены даже углом крена при выполнении разворота или виража, который не превышает 30 градусов.


    1. ivan01
      25.08.2016 12:52

      Абсолютно верно! А то приплели какие-то вихри на законцовках и якобы кроме них в экранном эффекте ничего нет, еще каких-то американских пилотов за уши притянули.


    1. Kekkai
      25.08.2016 13:56
      +1

      Большие пассажирские могут делать крен больше 30 градусов и «Бочку» они так же могут.
      На всем протяжении маневра сохраняется положительная перегрузка. Ограничение, разве что, в потребной высоте.


      1. ivan_petroff
        26.08.2016 05:43

        Крен для пассажирских самолетов в 30 градусов в развороте или вираже ограничен авиационными правилами во всем мире, например FAA (США). Далее у пассажиров будет выворачивать желудок наружу из-за увеличения перегрузки на пассажира за счет центростремительной силы самолета. Примерно так человек ощущает себя, когда его вдавливает скоростной лифт на начальном этапе движения вверх.
        При крене 45 градусов и более меняются местами рули управления самолета. Рулем направления ( управление влево — вправо) становится рулем высоты — поддерживается горизонтальное управления (вверх-вниз), а руль высоты становится рулем направления, что может привести к выходу самолета за пределы расчетных режимов полета и, как следствие, к катастрофе.

        Конструкция любого известного ныне пассажирского самолеты на фигуры пилотажа не рассчитана и максимум выдерживают перегрузку 3.5 g, а далее конструкция получает остаточные пластические деформации и затем разрушается.

        Для примера, попадание в зону турбулентности, особенно в грозовую, может создать перегрузку на конструкцию самолета более 3.5 g. Такой случай был с самолетом Як-40 Армянского управления Гражданской авиации СССР, который выдержал перегрузку 4.1. и получил остаточную деформацию центроплана, но смог завершить полет. Самолет был снят с эксплуатации. Нынешние самолеты такие нагрузки не выдержат.

        Советую автору статьи не закладывать в конструкцию самолета крен выполнение крена более 40 градусов.


        1. Kekkai
          26.08.2016 10:27

          Исходя из

          Самолеты для пилотажа специально проектируются и имеют ряд конструктивных отличий от пассажирских, последние ограничены даже углом крена при выполнении разворота или виража, который не превышает 30 градусов.

          Выходит, что ты говоришь о техническом, а не правовом ограничении. И правилами запрещено, да. Но не техникой.

          При выполнении «бочки» (или для пассажирского, скорее, «кадушки») перегрузка не достигнет более 2 единиц.
          Выполнение горизонтального разворота с креном 60 градусов транспортным Ан-178 проходит с перегрузкой около 2.2. А для поддержания горизонта перегрузка требуется больше.

          При крене 45 градусов и более меняются местами рули управления самолета. Рулем направления ( управление влево — вправо) становится рулем высоты — поддерживается горизонтальное управления (вверх-вниз), а руль высоты становится рулем направления, что может привести к выходу самолета за пределы расчетных режимов полета и, как следствие, к катастрофе.


          Если мы продолжаем говорить о больших самолетах, то органы управления не меняются местами. Эффективности руля направления недостаточно, чтобы управлять высотой и площади киля (а возможно и прочности) недостаточно для удержания высоты. Он используется для устранения скольжения на крыло, т.е. создания координированного разворота.

          Удержание горизонта при неизменном крене происходит за счет увеличения положительной перегрузки (ny), которую регулирует поворотный стабилизатор (руля высоты большому самолету будет недостаточно). И чем больше крен, тем больше нужна перегрузка.

          При «бочке» направление менять не нужно и удерживать горизонт, особо, тоже.


          1. ivan_petroff
            26.08.2016 11:47

            «Выходит, что ты говоришь о техническом, а не правовом ограничении. И правилами запрещено, да. Но не техникой.»

            Производитель самолета установил ограничение на крен самолета при развороте или вираже максимум 60-75 град в зависимости от типа гражданского самолета. На маневренных военных самолетах такие ограничения отсутствуют.

            «Если мы продолжаем говорить о больших самолетах, то органы управления не меняются местами...»

            Вы о каком крене пишете? Перегрузка на разворотах и виражах возникает за счет большей скорости полета необходимой для совершения маневра и соответственно большей необходимой подъемной силы, которая раскладывается на вертикальную для обеспечения постоянной высоты и центростремительную для обеспечения вращения при вираже. Разложите на бумаге и посмотрите внимательно.

            Перекладка стабилизатора совершается медленно на больших пассажирских самолетах и использовать ее проблематично при совершении даже простого маневра.


            1. Kekkai
              26.08.2016 13:42

              Производитель самолета установил ограничение на крен самолета при развороте или вираже максимум 60-75

              О каком производителе и типе идет речь?

              Вы о каком крене пишете? Перегрузка на разворотах и виражах возникает за счет большей скорости полета ...

              Если заложить крен хоть 10 градусов и не удерживать горизонт рулем высоты, то будет происходить снижение по спирали. И скорость соответственно будет расти, но перегрузка не изменится, а увеличится радиус разворота.

              Если парировать снижение рулем высоты, то скорость будет падать из-за возросшего сопротивления.
              Можно конечно руль высоты не трогать, а увеличить скорость изменив режим двигателям, но это опять же приведет к увеличению радиуса, а не росту перегрузки.

              Я думаю, что и при крене 90 можно удержаться в горизонте с нормальной перегрузкой, но потребная для этого скорость недостижима большими пассажирскими.

              Перегрузка в вираже возникает когда только скорость не позволяет удержаться в горизонте и приходится увеличивать угол атаки стабилизатором (или рулями высоты) чтобы увеличить подъемную силу крыла. На это конечно расходуется скорость.

              Перекладка стабилизатора совершается медленно на больших пассажирских самолетах и использовать ее проблематично при совершении даже простого маневра.

              Разворот с большим креном — это не очень простой маневр. И длится он десятки секунд. Поэтому скорости перекладки стабилизатора вполне достаточно. Пилоты изменяют его положение постоянно, иногда чаще чем рулей высоты.


              1. ivan_petroff
                26.08.2016 16:09

                «О каком производителе и типе идет речь?»

                Боинге. Аэйрбас не уточнял.

                «Я думаю, что и при крене 90 можно удержаться в горизонте с нормальной перегрузкой»

                Проекция подъемной силы самолета на вертикальную ось равна нулю в таком случае. Святой дух вам в помощь :-)


    1. encyclopedist
      25.08.2016 17:55

      Да будет вам известно, что большие пассажирские самолёты тоже нормально летают вверх ногами.


      Ну и ещё раз — симметричные профили тоже отлично работают.


      1. ivan_petroff
        26.08.2016 08:17

        «Да будет вам известно, что большие пассажирские самолёты тоже нормально летают вверх ногами.»

        Такой фокус был вполне возможен на самолете Ту-104 у которого прототипом был военный бомбардировщик, плюс главный конструктор самолете Егер ошибся при проектировании и заложил повышенную прочность конструкции, что снижало существенно экономическую эффективность, но прощало ошибки пилотов на посадке, которые приходили с поршневой авиации и частенько стучали самолетом о ВВП.

        «Ну и ещё раз — симметричные профили тоже отлично работают.»

        Симметричные профили неплохо работали на хвостовом оперении гражданских самолетов, но сейчас от них отказываются и используют обратно выпуклый профиль, чтобы обеспечить срыв потока на крыле при обледенении самолета таким образом, чтобы он наступал после срыва на крыле, а не наоборот, что приводило не раз к катастрофам, особенно в обледенении.

        На гражданских дозвуковых самолетах симметричные профили крыла не экономичны.


    1. Bronx
      27.08.2016 02:18
      +1

      > Экранный эффект вблизи поверхности (водной или земной) также хорошо описывается в соответствии с законом Бернулли. На малой высоте происходит торможения потока между нижней поверхностью профиля крыла и поверхностью экрана.

      Сможете описать, как из принципа Бернулли вывести торможение потока в сужающемся канале между крылом и экраном?


  1. vconst
    25.08.2016 12:54

    Вопрос дилетанта в аэродинамике
    Если под крылом образуется область с давлением выше атмосферного, а над крылом — ниже атмосферного, то значит к воздуху уже нельзя подходить, как к несжимаемой жидкости, так? А вот на сверхсзвуке, уже можно сравнивать воздух с жидкостью?


    1. 0serg
      25.08.2016 13:37

      В жидкости тоже могут быть области разного давления. Несжимаемость говорит только о том что попадая в такие области поток будет разгоняться / тормозиться, не меняя при этом свою плотность. На низких скоростях давления в потоке вокруг крыла слишком малы чтобы заметно сжать воздух, поэтому как раз для низких скоростей обтекание крыла воздухом как раз очень мало отличается от обтекания потоком идеальной жидкости. А на сверхзвуке усилия действующие на поток воздуха намного больше и поэтому как раз в таких режимах сжимаемость воздуха начинает играть заметную и важную роль.


      1. vconst
        25.08.2016 13:45

        Все я не так понял… Эх…

        Но тогда получается, если на дозвуке разница давлений не высока — то она не может играть значительную роль в подъемной силе?


        1. ivan01
          25.08.2016 13:55

          Нееет, это допустим 1% от атмосферного, параметры газа меняет мало, но 100Па на 10 м^2 это уже 100кгс ;)


          1. vconst
            25.08.2016 13:58

            Но возникает эта разница давлений, от угла атаки? А не от разницы в скорости потоков воздуха? Или разница в скоростях тоже имеет какое-то значение?


            1. 0serg
              25.08.2016 14:05

              В несжимаемом потоке не имеющем внешних источников энергии скорость и давление взаимно-однозначно связаны соотношением Бернулли. Поэтому строго говоря оба объяснения равнозначны (из изменения скоростей следует изменение давлений и наоборот, разница давлений порождает разницу скоростей). Остальное — дело вкуса. Я предпочитаю считать что скорость порождается разницей давлений, но, к примеру, в расчетах часто удобнее считать подъемную силу из разницы скоростей.

              Процитирую пример который я уже приводил выше. Представьте себе что мы взяли тележку и толкнули ее под гору. Скорость тележки и высота на которой она находится тоже связаны взаимно-однозначным соотношением. Поэтому формально можно сказать как «скорость тележки увеличивается за счет того что она катится под гору и теряет высоту» так и «тележка катится с горы потому что скорость ее увеличивается». Но субъективно мне кажется то первая формулировка лучше отражает происходящее.


            1. ivan01
              25.08.2016 14:07

              Смотрите, причинно следственная цепочка! )

              Угол атаки — геометрическая условность. Забудьте. Есть тело определенной формы, эта форма создает разные потоки воздуха сверху и снизу и разные поля давления соответственно. Вот это правда. Давление и скорость потока связанны уравнением Бернули. Это правда. Вязкость обеспечивает взаимодействие. Это правда. Все остальное спекуляция.


            1. vconst
              25.08.2016 14:13

              Спасибо, теперь понятнее стало